Question

Question 6 19 marks In this question, where necessary, you should round answers appropriately. (a) A journalist writing an ar

0 0
Add a comment Improve this question Transcribed image text
Answer #1

Please refer the below handwritten notes and requesting you to rate it with a like,it helps me alot.Thank you!!

As per the HOMEWORKLIB RULES we are allowed to solve only one subpart in a question,by keeping that a side,to help you I answered the two subparts in the question, I was begging you to considering it and please provide me with a like.

Please comment if you have any doubts.I beg you not to dislike,my account was in critical situation.All the best.

Add a comment
Know the answer?
Add Answer to:
Question 6 19 marks In this question, where necessary, you should round answers appropriately. (a) A...
Your Answer:

Post as a guest

Your Name:

What's your source?

Earn Coins

Coins can be redeemed for fabulous gifts.

Not the answer you're looking for? Ask your own homework help question. Our experts will answer your question WITHIN MINUTES for Free.
Similar Homework Help Questions
  • Question 1 - 20 marks This question is based on your work on MỤ123 up to and including Unit 10. K...

    Question 1 - 20 marks This question is based on your work on MỤ123 up to and including Unit 10. Katie likes to play volleyball. When she serves, the trajectory of the ball after she hits it can be modelled by the quadratic equation 295 where y represents the height in metres of the ball above the ground, and x represents the horizontal distance in metres of the ball from the position where it was struck by Katie. Assume that...

  • please do number 3 signment Submission For this assignment, you submit answers by question parts. The...

    please do number 3 signment Submission For this assignment, you submit answers by question parts. The number of submissions remaining for each question part only changes if you submit or change the answer signment Scoring Your last submission is used for your score 1 - 18 points Homework 6 1. Glven Y-C+IC-A + bYd, I = lo and Yd - YDT where A-100, - .6, lo = 40 and T-50 answer the following. a. Find the reduced form equation for...

  • Question 3 (covers Unit 6) - 25 marks . Your answers to part (a) of this question must be shown with your own work . P...

    Question 3 (covers Unit 6) - 25 marks . Your answers to part (a) of this question must be shown with your own work . Part (b) and (c) of this question are required you to use Mathcad to compute your solutions. You must submit a copy or attached a copy of your Mathcad solutions to your tutor for marking (a) Consider the following matrix 2 1 -2 A-0 5 0 (i) Find three eigenvalues λ of A. One of...

  • Question 5 [12 10 22 marks] (a) In a given inertial reference frame, S', consider a...

    Question 5 [12 10 22 marks] (a) In a given inertial reference frame, S', consider a region of space where there is a uniform and constant electric field, E', and zero magnetic field, i.e. B' = 0. The frame S' moves with respect to an observer, in another frame S, with velocity v. Write an expression for the electric field, E, observed in S? Clearly explain any notation (i) and new quantities introduced Write an expression for the magnetic field,...

  • please solve number one, two, and three Assignment Submission For this assignment, you submit answers by...

    please solve number one, two, and three Assignment Submission For this assignment, you submit answers by question parts. The number of submissions remaining for each question part only changes if you submit or change the answer Assignment Scoring Your last submission is used for your score 1 - 18 points Homework 6 1. Given Y-C+1, C-A + bYd, 1 - 1o and Yd - YOT where A-100, - .6, lo - 40 and T-50 answer the following. a. Find the...

  • Question 5 (Unit 6) - 31 marks (a) Express the following inhomogeneous system of first-order differential...

    Question 5 (Unit 6) - 31 marks (a) Express the following inhomogeneous system of first-order differential equations for x(t) and y(t) in matrix form: = 2x + y + 3e", y = 4x – y. Write down, also in matrix form, the corresponding homogeneous system of equations. (b) Find the eigenvalues of the matrix of coefficients and an eigenvector corresponding to each eigenvalue. (c) Hence write down the complementary function for the system of equations. (d) Find a particular integral...

  • Question 5 (Unit 6) - 31 marks (a) Express the following inhomogeneous system of first-order differential...

    Question 5 (Unit 6) - 31 marks (a) Express the following inhomogeneous system of first-order differential equations for x(t) and y(t) in matrix form: = 2x + y + 3e", y = 4x – y. Write down, also in matrix form, the corresponding homogeneous system of equations. (b) Find the eigenvalues of the matrix of coefficients and an eigenvector corresponding to each eigenvalue. (c) Hence write down the complementary function for the system of equations. (d) Find a particular integral...

  • Question C. I don' know where to start. I've looked at worked solutions to similar problems...

    Question C. I don' know where to start. I've looked at worked solutions to similar problems but I can't explain why each step was taken. 3. (a) Write down the integral form of Faraday's law (Maxwell's third equation) and briefly explain what is meant by each of the terms in the equation. (3 marks) (b) With reference to Faraday's law, what is meant by the terms motional emf and magnetic induction? (2 marks) (c) Consider a uniform magnetic field that...

  • This is equation 8: full question, which contains y1 ya(t) = n(e) / 102 . Use...

    This is equation 8: full question, which contains y1 ya(t) = n(e) / 102 . Use Equation (8) above or go through the reductio-of-order process to find a second solution 72 of the preceding equation such that {/1,} is a fundamental set of solutions of y" - (1+ y + 4y = 0 on (0,0). Y2 = Y = Solve the initial-value problem y"(t) + 4y'(t) + 13y(t) =0, y(0) = 3, y'(O) = 6. Express you answer in the...

  • Question 2: (26 marks) 2.1 Find the The Laplace transform of the following function t, if...

    Question 2: (26 marks) 2.1 Find the The Laplace transform of the following function t, if 03t<1 2t, if t1 [3] 2.2 Find the inverse Laplace transform of 10e 2 52 - 53 +632 - 25 + 5 (10] 2.3 Find y(4) if y(t) = u(t){t - 2)2 - us(t)/(t - 3) - 2) - us(t)e' (51 2.4 Solve the following initial value problem given by y" + 4y = 28.(t) (0)=1/(0) = 0 181 Question 3: (17 marks) Let...

ADVERTISEMENT
Free Homework Help App
Download From Google Play
Scan Your Homework
to Get Instant Free Answers
Need Online Homework Help?
Ask a Question
Get Answers For Free
Most questions answered within 3 hours.
ADVERTISEMENT
ADVERTISEMENT
ADVERTISEMENT